Halbzahliger Drehimpuls

Bedeutet halbzahliger Drehimpuls, dass sich das Teilchen immer dreht? Befindet sich beispielsweise ein Teilchen in a l = 1 2 Zustand, das heißt M = ± 1 2 und da L z = M F l M Das Teilchen dreht sich immer in der z-Komponente.

Verstehe ich das falsch? Wie kann sich ein Teilchen immer drehen? Ist das ein Perpetuum Mobile? Ich habe ein bisschen herumgesucht und sehe immer wieder, dass Teilchen eigentlich nur einen ganzzahligen Drehimpuls haben können. Anscheinend hat das etwas mit Normalisierung zu tun. Was ist los?

Wenn Sie über Quantenmechanik sprechen (da bin ich mir ziemlich sicher), dann hat Spin (soweit wir wissen) nichts mit Rotation zu tun.
Sehen Sie sich „The Story of Spin“ von Tomonaga an.

Antworten (2)

Teilchen haben einen halbzahligen Spin! Alle Fermionen tun es. Es sind die Bosonen, die einen ganzzahligen Spin haben müssen.

Der Spin ist ein Eigendrehimpuls eines Teilchens. Sie können versuchen, sich vorzustellen, dass sich das Teilchen dreht, wenn Ihnen das hilft, besser zu schlafen, aber das ist nicht wirklich das, was passiert.

Sie haben vielleicht gehört, dass der Drehimpuls aufgrund der Symmetrie unter Rotationen erhalten bleibt. In der Quantenmechanik wird dies noch extremer gemacht – der Drehimpuls charakterisiert lediglich, wie sich ein Quantenzustand unter Drehungen umwandelt. In diesem Sinne bedeutet die Aussage, dass ein Elektron Spin 1/2 hat, dass das Elektron nicht rotationssymmetrisch ist (aber es bedeutet nicht, dass es ein Ellipsoid oder so etwas ist!), sondern sich (quantenmechanisch!) Unter Drehungen im einfachsten Nicht umwandelt -Trivialer Weg möglich. Es stellt sich heraus, dass dies bedeutet, dass es aufgrund quantenmechanischer Gesetze auch einen Eigendrehimpuls hat. Der einzige Weg, es wirklich zu verstehen, besteht darin, die Quantenmechanik von Grund auf sorgfältig zu studieren.

Übrigens, M das Sie in der Frage verwenden, ist nur eine Projektion des Drehimpulses auf eine Achse. Selbst wenn Sie einen integralen Drehimpuls haben, "drehen" Sie sich immer noch mit diesem vollen Drehimpuls in eine Richtung, obwohl die Projektion Ihrer "Rotation" auf die orthogonale Achse Null sein kann.

Nehmen Sie ohne Beschränkung der Allgemeinheit an, dass der Drehimpulsoperator nur eine Komponente hat, L z ^ . Es kann gezeigt werden, dass die folgende Eigenwertgleichung gilt.

L z | l , M = M | l , M so dass M Z .

Das heißt, die Eigenwerte des Bahndrehimpulsoperators sind ganzzahlige Vielfache von . Sie können dies auf zwei Arten zeigen:

In der Koordinaten-(Polar-)Darstellung ist L z ^ = ich ϕ . Die Eigenwertgleichung,

L z ^ | l , M = l z | l , M ,

impliziert, dass die (unnormierte) Eigenfunktion von L z ^ Ist:

e ich l z ϕ .

Wir können das gewünschte Ergebnis beweisen durch:

  1. Auferlegen der Bedingung, dass die oben genannte Eigenfunktion einwertig sein muss; Und
  2. L z ^ ist ein hermitescher Operator. Das ist:
    ψ 1 | L z ^ | ψ 2 = ψ 2 | L z ^ | ψ 1

Wir können auch einen Drehimpuls haben, dessen Eigenwerte halbzahlige Vielfache davon sind . Dies wird als Spindrehimpuls bezeichnet. Man kann zeigen, dass in der matrixmechanischen Formulierung der Quantenmechanik die Eigenwerte des Drehimpulses im Allgemeinen sowohl halbe als auch ganze ganzzahlige Vielfache davon sein können .

Man sollte sich den Spin-Drehimpuls nicht als klassisches Gegenstück vorstellen. Lassen Sie mich Ihnen ein Beispiel geben:

Wenden Sie den Rotationsoperator (unabhängig von seiner funktionalen Form) auf einen Quantenzustand an, dessen Spin ins Positive zeigt z Richtung. Das Ergebnis ist folgendes:

R ^ ( 2 π ) | + z = | + z ,

wo ich mich entschieden habe, den Spin-Zustand um 360 Grad zu drehen. Entgegen der klassischen Intuition ist der Zustand beispielsweise eines Elektrons nicht das, was wir erwarten: Klassischerweise erwarten wir, dass das Ergebnis der Anwendung des Rotationsoperators dasselbe ist wie das Ergebnis unserer Anwendung des Identitätsoperators, was das ergibt Anfangszustand als Endzustand. Es dreht sich nur, wenn man die Elektronen durchdreht 4 π Radianten, dass End- und Anfangszustand gleich sind. Das Ergebnis ist sowohl kontraintuitiv als auch gegen die klassische Intuition.

Hinweis: Um pedantisch zu sein, die Quantenzustände auf der rechten und linken Seite sind äquivalent/gleich. (warum?) Also ignorieren Sie bitte die Terminologie in ihrer korrekten QM-Interpretation, da ich im vorherigen Absatz umwandeln möchte, dass die Ausdrücke auf der rechten und linken Seite nicht übereinstimmen.

Daher sollte man keine Entsprechung zwischen dem Drehimpuls des Spins und klassischer Intuition/Ergebnissen herstellen. So kann beispielsweise der Spin eines Elektrons auf einer Blochkugel vollständig angegeben werden und man verwendet üblicherweise den Begriff, dass man auf der Blochkugel „Rotationen“ ausführen kann, die man sich (zum Beispiel) als Ergebnis der Zeitentwicklung vorstellen kann eine Bloch-Kugel. Ja, das können wir, aber man sollte sich diese "Rotationen" nicht als Operationen vorstellen, die in allen Fällen ein klassisches Gegenstück haben, wie zB im vorigen Beispiel. Daher ist der interessierende Quantenoperator in diesem Fall normalerweise kein Rotationsoperator, sondern ein unitärer Operator, der als Verallgemeinerung eines Rotationsoperators in entweder einem komplexen Vektorraum oder einem Hilbert-Raum angesehen werden kann.

Es ist irreführend zu sagen, dass der Spin nach einer 360-Grad-Drehung "umgedreht" ist. Sie sagen später, dass die Zustände äquivalent sind, aber "umgedreht" kann von jemandem mit einer tatsächlichen Richtungsänderung des Spins verwechselt werden. Ich sage das nur, weil ich oft auf dieses Missverständnis stoße :)
Ja, ich habe den Beitrag bearbeitet, um die oben genannte Mehrdeutigkeit zu beseitigen.
Nitpicking: Es gibt noch einen Teil, der sagt "... dass der Spin gleich bleiben würde". Ich persönlich denke nicht, dass irgendeine klassische Intuition verletzt wird, da es keine klassische Intuition für Zustandsvektoren gibt; nur für den Zustand, der sich als Strahl (bis auf eine Phase definierter Vektor) so verhält, wie man es klassisch erwarten würde. Aus diesem Grund lassen wir überhaupt projektive Darstellungen zu. Auf jeden Fall scheint es für die vorliegende Frage, die sich hauptsächlich mit der Idee des Eigendrehimpulses befasst, nicht sehr relevant zu sein.
Ja stimmt. Obwohl ich selbst im Kontext von SO (3) und SU (2) kein sehr tiefes Verständnis für projektive Darstellungen habe, habe ich anfangs darüber nachgedacht, ob ich in der Antwort einen Kommentar in diese Richtung hinzufügen soll oder nicht, aber dann habe ich mich entschieden dies nicht zu tun, da der Umfang der Frage eindeutig anders ist.